Difference between revisions of "2017 AMC 12A Problems/Problem 6"
(Created page with "Joy has <math>30</math> thin rods, one each of every integer length from <math>1 \text{ cm}</math> through <math>30 \text{ cm}</math>. She places the rods with lengths <math>3...") |
(Added See Also) |
||
Line 1: | Line 1: | ||
+ | ==Problem== | ||
+ | |||
Joy has <math>30</math> thin rods, one each of every integer length from <math>1 \text{ cm}</math> through <math>30 \text{ cm}</math>. She places the rods with lengths <math>3 \text{ cm}</math>, <math>7 \text{ cm}</math>, and <math>15 \text{cm}</math> on a table. She then wants to choose a fourth rod that she can put with these three to form a quadrilateral with positive area. How many of the remaining rods can she choose as the fourth rod? | Joy has <math>30</math> thin rods, one each of every integer length from <math>1 \text{ cm}</math> through <math>30 \text{ cm}</math>. She places the rods with lengths <math>3 \text{ cm}</math>, <math>7 \text{ cm}</math>, and <math>15 \text{cm}</math> on a table. She then wants to choose a fourth rod that she can put with these three to form a quadrilateral with positive area. How many of the remaining rods can she choose as the fourth rod? | ||
<math>\textbf{(A)}\ 16 \qquad\textbf{(B)}\ 17 \qquad\textbf{(C)}\ 18 \qquad\textbf{(D)}\ 19 \qquad\textbf{(E)}\ 20</math> | <math>\textbf{(A)}\ 16 \qquad\textbf{(B)}\ 17 \qquad\textbf{(C)}\ 18 \qquad\textbf{(D)}\ 19 \qquad\textbf{(E)}\ 20</math> | ||
+ | |||
+ | ==Solution== | ||
+ | |||
+ | ==See Also== | ||
+ | {{AMC12 box|year=2017|ab=A|num-b=5|num-a=7}} |
Revision as of 14:50, 8 February 2017
Problem
Joy has thin rods, one each of every integer length from through . She places the rods with lengths , , and on a table. She then wants to choose a fourth rod that she can put with these three to form a quadrilateral with positive area. How many of the remaining rods can she choose as the fourth rod?
Solution
See Also
2017 AMC 12A (Problems • Answer Key • Resources) | |
Preceded by Problem 5 |
Followed by Problem 7 |
1 • 2 • 3 • 4 • 5 • 6 • 7 • 8 • 9 • 10 • 11 • 12 • 13 • 14 • 15 • 16 • 17 • 18 • 19 • 20 • 21 • 22 • 23 • 24 • 25 | |
All AMC 12 Problems and Solutions |